67-1-12 Forum

Prepare for the LSAT or discuss it with others in this forum.
Post Reply
User avatar
sdwarrior403

Bronze
Posts: 114
Joined: Fri Aug 10, 2012 8:13 pm

67-1-12

Post by sdwarrior403 » Wed Nov 28, 2012 7:57 pm

I had it between b and d. I see really no difference between the two.

User avatar
sdwarrior403

Bronze
Posts: 114
Joined: Fri Aug 10, 2012 8:13 pm

Re: 67-1-12

Post by sdwarrior403 » Wed Nov 28, 2012 9:02 pm

It seems like both answer choices directly support the idea that insufficient dosage was the issue.

B does this by saying that the chance of the beverage being the issue has been lessened.

D does this by saying that the beverage is not the only cause. I felt that, if anything, D was kind of a bad answer as it suggests that the beverage could still be the cause.

User avatar
ScottRiqui

Gold
Posts: 3633
Joined: Mon Nov 29, 2010 8:09 pm

Re: 67-1-12

Post by ScottRiqui » Wed Nov 28, 2012 9:33 pm

Here are the steps in the patient's treatment:

Single dose and drinking beverage - still had symptoms.
**doctor makes first set of recommendations**
Single dose and stop drinking beverage - still had symptoms
**doctor makes second set of recommendations**
Double dose and still no beverage - symptoms gone.



Answer B doesn't really address whether or not the beverage contributed to the ineffectiveness of the treatment one way or the other, because the patient had already stopped drinking the beverage before the doctor's second set of recommendations (double the dosage and continue not taking the beverage). The only thing that changed as a result of the doctor's second set of recommendations was a doubling of the dosage.

Answer D is correct, because before the second recommendation was implemented, you had "no tea" and "normal dosage". After the recommendation was implemented, you had "no tea" and "double dosage". Since the patient's symptoms went away with a double dose this time (unlike earlier when the patient was still drinking the beverage), that suggests that the beverage wasn't the only reason that a normal dose didn't work the first time. Maybe the patient is drug-resistant, maybe the medicine in question is old or was made improperly - who knows?

User avatar
sdwarrior403

Bronze
Posts: 114
Joined: Fri Aug 10, 2012 8:13 pm

Re: 67-1-12

Post by sdwarrior403 » Wed Nov 28, 2012 10:09 pm

ScottRiqui wrote:

Answer B doesn't really address whether or not the beverage contributed to the ineffectiveness of the treatment one way or the other, because the patient had already stopped drinking the beverage before the doctor's second set of recommendations (double the dosage and continue not taking the beverage).
I believe you are incorrect on the bolded part. The second recommendation was actually the doctor telling the patient to stop drinking the beverage and take the initial dosage. So it is not true that the second recommendation involves doubling the dosage.

Given that this is incorrect in your explanation, I would love a revision of your explanation, as I am still in the dark as to the distinction between B and D.

User avatar
Cobretti

Gold
Posts: 2593
Joined: Tue Aug 21, 2012 12:45 am

Re: 67-1-12

Post by Cobretti » Wed Nov 28, 2012 10:25 pm

sdwarrior403 wrote:It seems like both answer choices directly support the idea that insufficient dosage was the issue.

B does this by saying that the chance of the beverage being the issue has been lessened.

D does this by saying that the beverage is not the only cause. I felt that, if anything, D was kind of a bad answer as it suggests that the beverage could still be the cause.
The evidence doesn't show it was less likely. It shows it was a cause because the double dosage + tea had no effect, but double dosage + no tea worked. D is correct.

Your problem is that you didn't get his argument. His argument showed that it was the beverage AND the dosage that were problems, which in the process supported his hypothesis that the dosage was insufficient. He never said the dosage was the only problem.

I also think you slightly misread the question stem. Its not saying "which of the following would support his hypothesis", its asking what his argument actually said.
sdwarrior403 wrote:
ScottRiqui wrote:

Answer B doesn't really address whether or not the beverage contributed to the ineffectiveness of the treatment one way or the other, because the patient had already stopped drinking the beverage before the doctor's second set of recommendations (double the dosage and continue not taking the beverage).
I believe you are incorrect on the bolded part. The second recommendation was actually the doctor telling the patient to stop drinking the beverage and take the initial dosage. So it is not true that the second recommendation involves doubling the dosage.

Given that this is incorrect in your explanation, I would love a revision of your explanation, as I am still in the dark as to the distinction between B and D.
The first recommendation was the initial dosage. The second recommendation was double dosage + tea.

Want to continue reading?

Register now to search topics and post comments!

Absolutely FREE!


User avatar
sdwarrior403

Bronze
Posts: 114
Joined: Fri Aug 10, 2012 8:13 pm

Re: 67-1-12

Post by sdwarrior403 » Wed Nov 28, 2012 10:30 pm

mrizza wrote:
The first recommendation was the initial dosage. The second recommendation was double dosage + tea.
That's a distortion. The first recommendation is clearly the doubling of the dosage. The second is the intial dosage (not double) and stop tea. The third is the double and stop tea.

User avatar
Cobretti

Gold
Posts: 2593
Joined: Tue Aug 21, 2012 12:45 am

Re: 67-1-12

Post by Cobretti » Wed Nov 28, 2012 10:34 pm

sdwarrior403 wrote:
mrizza wrote:
The first recommendation was the initial dosage. The second recommendation was double dosage + tea.
That's a distortion. The first recommendation is clearly the doubling of the dosage. The second is the intial dosage (not double) and stop tea. The third is the double and stop tea.
You're wrong.

It says "...describes the manner in which the doctor's second set of reccommendations..."

ETA: So everything after the intial dosage was the second set that it is asking about. to include double dosage + tea, initial dosage + no tea, and double dosage + no tea

User avatar
ScottRiqui

Gold
Posts: 3633
Joined: Mon Nov 29, 2010 8:09 pm

Re: 67-1-12

Post by ScottRiqui » Wed Nov 28, 2012 10:38 pm

I don't think that the initial advice to simply double the dosage counts as a "set of recommendations".

But even if the second set of recommendations was in fact to quit drinking the beverage and resume the normal dosage, answer D is still correct. You've eliminated the beverage and administered the normal dose, but the patient still has symptoms. That suggests that there's some other factor besides the beverage that's keeping the normal dose of medication from being effective, and that's exactly what answer D is saying.

But I can see why you can make an argument for B as well - if removing the beverage still doesn't alleviate the symptoms, I would think that WOULD make it less plausible that the beverage was causing the ineffectiveness. I still think that D is the better answer under your interpretation of the question, but I realize even though LSAT is asking for the answer that "most accurately describes...", there should only be one "right" answer, not two answers with varying degrees of correctness.

In the end, I still think that the second SET of recommendations was to double the dose and to continue not drinking the beverage. That makes more sense considering that "set of recommendations" is undeniably a plural term, plus it also makes answer D correct and answer B wrong, eliminating the apparent problem of having two arguably "correct" answers.

User avatar
Cobretti

Gold
Posts: 2593
Joined: Tue Aug 21, 2012 12:45 am

Re: 67-1-12

Post by Cobretti » Wed Nov 28, 2012 10:41 pm

ScottRiqui wrote:I don't think that the initial advice to simply double the dosage counts as a "set of recommendations".

But even if the second set of recommendations was in fact to quit drinking the beverage and resume the normal dosage, answer D is still correct. You've eliminated the beverage and administered the normal dose, but the patient still has symptoms. That suggests that there's some other factor besides the beverage that's keeping the normal dose of medication from being effective, and that's exactly what answer D is saying.

But I can see why you can make an argument for B as well - if removing the beverage still doesn't alleviate the symptoms, I would think that WOULD make it less plausible that the beverage was causing the ineffectiveness. I still think that D is the better answer under your interpretation of the question, but I realize even though LSAT is asking for the answer that "most accurately describes...", there should only be one "right" answer, not two answers with varying degrees of correctness.

In the end, I still think that the second SET of recommendations was to double the dose and to continue not drinking the beverage. That makes more sense considering that "set of recommendations" is undeniably a plural term, plus it also makes answer D correct and answer B wrong, eliminating the apparent problem of having two arguably "correct" answers.
There is no grey area for B. Double Dosage + tea = didn't work. Double Dosage + no tea = worked. He showed that it was one of the reasons.

Want to continue reading?

Register for access!

Did I mention it was FREE ?


natashka85

Bronze
Posts: 249
Joined: Thu Nov 01, 2012 4:29 pm

Re: 67-1-12

Post by natashka85 » Wed Nov 28, 2012 10:44 pm

Ok guys,D is the right answer it directly addresses the issue there that the initial dosage was not enough apart from the herbal remedy that he took,so D it has to support the initial hypothesis D directly does that which is the dosage not the herbal remedy,B is wrong cause it doesnt address the dosage issue.

User avatar
sdwarrior403

Bronze
Posts: 114
Joined: Fri Aug 10, 2012 8:13 pm

Re: 67-1-12

Post by sdwarrior403 » Wed Nov 28, 2012 10:45 pm

mrizza wrote:
You're wrong.

It says "...describes the manner in which the doctor's second set of reccommendations..."

ETA: So everything after the intial dosage was the second set that it is asking about. to include double dosage + tea, initial dosage + no tea, and double dosage + no tea
I think there is an ambiguity between a prescription given and then multiple recommendations given after that prescription: I do not believe it is fair to call prescribing a recommendation in that manner when the stimulus does not phrase it as such.

User avatar
sdwarrior403

Bronze
Posts: 114
Joined: Fri Aug 10, 2012 8:13 pm

Re: 67-1-12

Post by sdwarrior403 » Wed Nov 28, 2012 10:47 pm

natashka85 wrote:Ok guys,D is the right answer it directly addresses the issue there that the initial dosage was not enough apart from the herbal remedy that he took,so D it has to support the initial hypothesis D directly does that which is the dosage not the herbal remedy,B is wrong cause it doesnt address the dosage issue.
Sure B does. It makes the dosage issue more plausible due to the beverage being less plausible.

User avatar
ScottRiqui

Gold
Posts: 3633
Joined: Mon Nov 29, 2010 8:09 pm

Re: 67-1-12

Post by ScottRiqui » Wed Nov 28, 2012 10:50 pm

mrizza wrote:
sdwarrior403 wrote:
mrizza wrote:
The first recommendation was the initial dosage. The second recommendation was double dosage + tea.
That's a distortion. The first recommendation is clearly the doubling of the dosage. The second is the intial dosage (not double) and stop tea. The third is the double and stop tea.
You're wrong.

It says "...describes the manner in which the doctor's second set of reccommendations..."

ETA: So everything after the intial dosage was the second set that it is asking about. to include double dosage + tea, initial dosage + no tea, and double dosage + no tea
I read it a different way - the doctor initially made a single recommendation - double the dosage. Then he began making recommendations in sets - first to revert to the initial dosage and stop drinking the tea, and second, to double the dosage and continue not drinking the tea.

But I like your interpretation better, although I still wouldn't have counted the initial prescription as the first recommendation - the author even used "first" when talking about the recommendation to double the dose. Plus, we don't know that the doctor in the problem was even the one who initially prescribed the medicine for the patient in the first place - maybe the doctor was seeing the patient via referral because the normal course of treatment wasn't working?

After looking at the problem for a while longer, I think that doubling the dose was the initial recommendation, and "regular dose and no tea" and "double dose and no tea", taken together, make up the second set of recommendations. This interpretation both makes D right and B unequivocally wrong.

Register now!

Resources to assist law school applicants, students & graduates.

It's still FREE!


User avatar
sdwarrior403

Bronze
Posts: 114
Joined: Fri Aug 10, 2012 8:13 pm

Re: 67-1-12

Post by sdwarrior403 » Wed Nov 28, 2012 11:12 pm

ScottRiqui wrote: I read it a different way - the doctor initially made a single recommendation - double the dosage. Then he began making recommendations in sets - first to revert to the initial dosage and stop drinking the tea, and second, to double the dosage and continue not drinking the tea.

But I like your interpretation better, although I still wouldn't have counted the initial prescription as the first recommendation - the author even used "first" when talking about the recommendation to double the dose. Plus, we don't know that the doctor in the problem was even the one who initially prescribed the medicine for the patient in the first place - maybe the doctor was seeing the patient via referral?

After looking at the problem for a while longer, I think that doubling the dose was the initial recommendation, and "regular dose and no tea" and "double dose and no tea", taken together, make up the second set of recommendations. This interpretation both makes D right and B unequivocally wrong.
I agree with every word of this post.

I obviously agree that if you believe the second set of recommendations = "regular dose and no tea" + "double dose and no tea."

However, if you do not believe the second set includes those two, you are left with more of a problem in dismissing B.

mrizza is absolutely wrong in the interpretation of the prescription being the first recommendation.

User avatar
Cobretti

Gold
Posts: 2593
Joined: Tue Aug 21, 2012 12:45 am

Re: 67-1-12

Post by Cobretti » Thu Nov 29, 2012 12:02 am

sdwarrior403 wrote:
ScottRiqui wrote: I read it a different way - the doctor initially made a single recommendation - double the dosage. Then he began making recommendations in sets - first to revert to the initial dosage and stop drinking the tea, and second, to double the dosage and continue not drinking the tea.

But I like your interpretation better, although I still wouldn't have counted the initial prescription as the first recommendation - the author even used "first" when talking about the recommendation to double the dose. Plus, we don't know that the doctor in the problem was even the one who initially prescribed the medicine for the patient in the first place - maybe the doctor was seeing the patient via referral?

After looking at the problem for a while longer, I think that doubling the dose was the initial recommendation, and "regular dose and no tea" and "double dose and no tea", taken together, make up the second set of recommendations. This interpretation both makes D right and B unequivocally wrong.
I agree with every word of this post.

I obviously agree that if you believe the second set of recommendations = "regular dose and no tea" + "double dose and no tea."

However, if you do not believe the second set includes those two, you are left with more of a problem in dismissing B.

mrizza is absolutely wrong in the interpretation of the prescription being the first recommendation.
Good luck on sat

User avatar
sdwarrior403

Bronze
Posts: 114
Joined: Fri Aug 10, 2012 8:13 pm

Re: 67-1-12

Post by sdwarrior403 » Thu Nov 29, 2012 12:24 am

Another incorrect assumption you have made. How many can you make in one thread?

Get unlimited access to all forums and topics

Register now!

I'm pretty sure I told you it's FREE...


Post Reply

Return to “LSAT Prep and Discussion Forum”